1answer.
Ask question
Login Signup
Ask question
All categories
  • English
  • Mathematics
  • Social Studies
  • Business
  • History
  • Health
  • Geography
  • Biology
  • Physics
  • Chemistry
  • Computers and Technology
  • Arts
  • World Languages
  • Spanish
  • French
  • German
  • Advanced Placement (AP)
  • SAT
  • Medicine
  • Law
  • Engineering
posledela
3 years ago
6

a stone is vertically thrown upward with the velocity of 72km/hr find the maximum height reached the height​

Physics
1 answer:
dolphi86 [110]3 years ago
3 0

Answer:

40m

Explanation:

v^2=gh

h=v^2/g

h=20^2/10

h=400/10

h=40

You might be interested in
The Robinson projection map is considered very useful because
Vladimir79 [104]
Hello,
   The question states: <span>The Robinson projection map is considered very useful because...
The answer is \boxed{because \ most \ distances, \ sizes, \ and \ shapes \ are \ accurate.}

Hope this helped!

~FoodJunky
</span>
7 0
3 years ago
The mass of a particular eagle is twice that of a hunted pigeon. Suppose the pigeon is flying north at ????????,2=17.9vi,2=17.9
AleksAgata [21]

Answer:

V=27.24 m/s

Explanation:

We need to apply the linear momentum conservation theorem:

m_1*v_{o1}+m_2*v_{o2}=m_t*v_{f}\\

The velocity of the eagle its defined by its two components:

v_x=V*cos(\theta)\\v_y=V*sin(\theta)\\v_x=35.9*cos(61.9^o)=16.9m/s\\v_y=35.9*sin(61.9^o)=31.7m/s

2*(16.9m/s(i)+31.66m/s(j))+17.9m/s(i)=3*v_f\\v_f=17.23m/s(i)+21.10m/s(j)

because speed is a scalar value:

V=\sqrt{(21.10m/s)^2+(17.23)^2}\\V=27.24m/s

6 0
4 years ago
Light passes through a pair of very thin parallel slits. The resulting interference pattern is viewed far from the slits at vari
butalik [34]

Answer:

Intensity of the next bright fringe will remain same.

Explanation:

The question is based on Young's double slit experiment, since its about bright fringe, the interference here is constructive.

Young's condition for constructive interference is given by:

dsin\theta = n\lambda

where,

d = slits distance from eachother or width of the slits

\lambda = wavelength

n = interferance order

Also, we know that in Young's experiment, the fringe intensity is given by:

I' = 4Icos^{2}\phi

where,

\phi = phase difference

Therefore, in absence of phase difference i.e., \phi = 0, the intensity of the next bright fringe will not change and it will remain same.

4 0
3 years ago
Electrical energy is converted into kinetic energy in
Vanyuwa [196]
The answer would be a motor. If thats one of the options, it was for me :)
6 0
4 years ago
What is the change in air pressure a good indicator of a change in temperature be a change in the weather I decrease in humidity
Elza [17]

Answer:

A

Explanation:

warm air rises creating low presure

cold air sinks creating high presure

so this means that temprature is realated to presure.

3 0
3 years ago
Other questions:
  • Which statement best describes the skier?
    9·2 answers
  • Your first day on the job, you get a call from the owner complaining that her network connection is down. A quick check of the c
    15·1 answer
  • By what factor must the amplitude of a sound wave be increased in order to increase the intensity by a factor of 9?a. 9 b. 2 c.
    8·2 answers
  • Which of the following is not a part of the appendicular skeleton
    10·1 answer
  • Emboldened by the success of their late night keg pull in Exercise 61 above, our intrepid young scholars have decided to pay hom
    14·1 answer
  • How come the arrow points the other direction when we look at it through a glass of water?
    14·1 answer
  • What is the current that reverses direction in a regular patter called
    7·1 answer
  • Helium gas is compressed by an adiabatic compressor from an initial state of 14 psia and 50°F to a final temperature of 320°F in
    11·1 answer
  • 1. What does the term monecious mean?
    13·1 answer
  • 3. What do we call the ONLY part of the electromagnetic spectrum that we can
    6·1 answer
Add answer
Login
Not registered? Fast signup
Signup
Login Signup
Ask question!